(Differential equation) Finding an exponential equation

Click For Summary
SUMMARY

The discussion focuses on modeling the leak of oil from a 44-gallon barrel using a differential equation. The mathematical model is established as \(\frac{\partial{A}}{\partial{t}} = -Akt\), indicating that the rate of change of oil is proportional to both the time elapsed and the amount of oil present. The general solution derived is \(A(t) = Ce^{-\frac{kt^2}{2}}\), with the particular solution for the initial condition \(A(0) = 44\) resulting in \(A(t) = 44e^{-\frac{kt^2}{2}}\). The solutions provided are confirmed to be correct based on the context of the problem.

PREREQUISITES
  • Understanding of differential equations
  • Familiarity with exponential functions
  • Knowledge of initial value problems
  • Basic calculus, specifically integration techniques
NEXT STEPS
  • Study the application of differential equations in real-world scenarios
  • Learn about initial value problems and their solutions
  • Explore the concept of proportionality in rates of change
  • Investigate advanced techniques for solving nonlinear differential equations
USEFUL FOR

Students studying differential equations, mathematicians, and anyone involved in modeling dynamic systems such as fluid dynamics or resource depletion.

namegoeshere
Messages
5
Reaction score
0

Homework Statement



A 44 gallon barrel of oil develops a leak at the bottom. Let A(t) be the amount of oil in the barrel at a given time t. Suppose that the amount of oil is decreasing at a rate proportional to the product of the time elapsed and the amount of oil present in the barrel.

a. Give the mathematical model for A
b. Find the general solution of the differential equation
c. Find the particular solution for the initial condition

Homework Equations


The Attempt at a Solution


a.
\frac{\partial{A}}{\partial{t}} = -Akt​
b.
\int{\frac{1}{A}\,dA} = \int{-kt\,dt}
\ln{A}=\frac{kt^2}{2}+C
A(t)=Ce^\frac{-kt^2}{2}​
c.
44=Ce^0
A(t)=44e^\frac{-kt^2}{2}​

Is this correct? I'm not sure if I came up with the right \frac{\partial{A}}{\partial{t}}.
 
Last edited:
Physics news on Phys.org
All looks right to me.
 
Question: A clock's minute hand has length 4 and its hour hand has length 3. What is the distance between the tips at the moment when it is increasing most rapidly?(Putnam Exam Question) Answer: Making assumption that both the hands moves at constant angular velocities, the answer is ## \sqrt{7} .## But don't you think this assumption is somewhat doubtful and wrong?

Similar threads

  • · Replies 1 ·
Replies
1
Views
1K
Replies
2
Views
2K
  • · Replies 2 ·
Replies
2
Views
1K
  • · Replies 7 ·
Replies
7
Views
2K
Replies
4
Views
1K
  • · Replies 7 ·
Replies
7
Views
2K
  • · Replies 2 ·
Replies
2
Views
1K
  • · Replies 3 ·
Replies
3
Views
1K
  • · Replies 6 ·
Replies
6
Views
1K
Replies
13
Views
2K